Answer:
9.254
Step-by-step explanation:
Move the decimal three times (how many zeros of 1000 there is)
is 22π a whole number
Answer:
No it is irrational number.
Step-by-step explanation:
because 2pie cannot be expressed as a quotient of interger.
A school is having a pizza party. Each pizza is cut into 12 slices. Which explains whether it is better to estimate a greater or lesser number than the actual number of people that will eat pizza?
A.
It is better to estimate a lesser number of people so the school can save money by ordering fewer pizzas.
B.
It is better to estimate a greater number of people to make sure there is enough pizza for everyone.
C.
It is better to estimate a lesser number of people to make sure there will not be extra pizza that will get thrown away.
D.
It is better to estimate a greater number of people because they can return any extra pizza that does not get eaten.
Answer: I think B
Step-by-step explanation:
Answer:
It is better to estimate a greater number of people to make sure there is enough pizza for everyoneWhen baking soda and vinegar react, the surface bubbles. What does this most likely indicate?
a chemical change, because a precipitate is being formed
a chemical change, because a gas is being formed
a non-chemical change, because a precipitate is being formed
a non-chemical change, because a gas is being formed
Answer:
the answer is B
Lea wrote a number riddle her clues were : 9 is 10 times the value of 9 329,513
2 is 2/10 The value of 2 in 7,562
6 is 6/10 the value In 4.68 which is the answer ?
A) 19,426
B)95,862
C) 341,959.26
D) 498,173.26
Answer:
D) 498,173.26Step-by-step explanation:
1) 9 is 10 times the value of 9 in 329,513
9 is 1000'th place10*1000*9 = 90000The number contains 900002) 2 is 1/10 the value of 2 in 7,562
2 is one's place1/10*1*2 = 0.2The number contains 0.23) 6 is 1/10 the value of 6 In 4.68
6 is one tenths place1/10*0.1*6 = 0.06The number contains 0.06Sum of the three above:
90000 + 0.2 + 0.06 = 90000.26The answer option containing the above number is:
D) 498,173.26Negative numbers are _______________ less than positive numbers.
Always
Sometimes
Never
Answer:
sometimes
Step-by-step explanation:
Answer:
always
Step-by-step explanation:
find the equation of the line which passes through the point (7,2) and is perpendicular to y=5x-2
Answer:
[tex]\displaystyle y=-\frac{1}{5}x+\frac{17}{5}[/tex]
Step-by-step explanation:
Equation of a line
A line can be represented by an equation of the form
[tex]y=mx+b[/tex]
Where x is the independent variable, m is the slope of the line, b is the y-intercept and y is the dependent variable.
We need to find the equation of the line passing through the point (7,2) and is perpendicular to the line y=5x-2.
Two lines with slopes m1 and m2 are perpendicular if:
[tex]m_1.m_2=-1[/tex]
The given line has a slope m1=5, thus the slope of our required line is:
[tex]\displaystyle m_2=-\frac{1}{m_1}=-\frac{1}{5}[/tex]
The equation of the line now can be expressed as:
[tex]\displaystyle y=-\frac{1}{5}x+b[/tex]
We need to find the value of b, which can be done by using the point (7,2):
[tex]\displaystyle 2=-\frac{1}{5}*7+b[/tex]
Operating:
[tex]\displaystyle 2=-\frac{7}{5}+b[/tex]
Multiplying by 5:
[tex]10=-7+5b[/tex]
Operating:
[tex]10+7=5b[/tex]
Solving for b:
[tex]\displaystyle b=\frac{17}{5}[/tex]
The equation of the line is:
[tex]\boxed{\displaystyle y=-\frac{1}{5}x+\frac{17}{5}}[/tex]
7w-6 = 3(w + 6)
Please help me with this answer.
Answer:
w = 6
Step-by-step explanation:
Distribute the 3 first
7w - 6 = 3w + 18
Combine like terms
4w = 24
Divide by 4 to isolate w
so w = 6
Answer:
Step-by-step explanation:
Apply the distributive property to the right side of the equation. 3(w+6). Then, begin to simplify the equation. W=6
In minnesota,the state sales tax rate is 6.875%
The price tag on an HDTV gives the price as $560. There are no discounts available.
What is the total cost for the TV, including sales tax, in Minnesota?
Answer:
598.50
Step-by-step explanation:
Multiply the percent by the price then add that number to the price.
I added 38.50 in taxes to the price of the TV
The total cost of the TV will be $598.50 including the sales tax of 6.875%.
What is an expression?The mathematical expression combines numerical variables and operations denoted by addition, subtraction, multiplication, and division signs.
Mathematical symbols can be used to represent numbers (constants), variables, operations, functions, brackets, punctuation, and grouping. They can also denote the logical syntax's operation order and other properties.
Given that in Minnesota, the state sales tax rate is 6.875%. The price tag on an HDTV gives the price as $560,
The total cost will be calculated as,
Cost = 560 x 1.06875
Cost = 598.5
Hence, the total cost will be $598.50.
To know more about an expression follow
https://brainly.com/question/20879845
#SPJ2
slope=8, y-intercept=-4
Answer:
Y= 8X + 4
Step-by-step explanation:
One circle labeled right 28 overlaps another circle labeled female 7. The overlap is labeled 24. A 4-column table with 3 rows. The first column has no label with entries female, male, total. The second column is labeled right with entries 24, c, 52. The third column is labeled right with entries a, 6, d. The fourth column is labeled total with entries b, 34, d. A group of 65 baseball players were surveyed about which hand they favor for batting. The data from the survey are shown in the Venn diagram. Determine the value for each variable in the two-way table. a = b = c = d = e =
Answer:
a=7
b=31
c=28
d=13
e=65
Step-by-step explanation:
this is the right answer.
Answer:
a=7
b=31
c=28
d=13
e=65
Step-by-step explanation:
What is the value of F in this equation
F^2=9/49
Answer:
[tex] \sqrt{f {}^{2} } = \sqrt{ \frac{9}{49} } [/tex]
[tex]f = \frac{ \sqrt{9} }{ \sqrt{49} } [/tex]
[tex]f = \frac{3}{7} [/tex] or -3/7
Answer:
[tex]f=-\frac{3}{7},\frac{3}{7}[/tex]
Step-by-step explanation:
[tex]f^2=\frac{9}{49}[/tex]
Square root both sides of the equation:
[tex]\sqrt{f^2} =\sqrt{\frac{9}{49} }[/tex]
The square root of [tex]f^2[/tex] is [tex]f[/tex], so let's rewrite the equation:
[tex]f=\sqrt{\frac{9}{49} }[/tex]
Now, we can make this easier on ourselves by noticing that both the numerator and denominator of the fraction are square numbers, so rewrite the equation:
[tex]f=\frac{\sqrt{9} }{\sqrt{49}}[/tex]
Find the square root of 9 (-3 & 3), and the square root of 49 (-7 & 7). Due to the possibility of the positive or negative solution, there will be two:
[tex]f=-\frac{3}{7} ,\frac{3}{7}[/tex]
Two points,L and N are 160 m apart. M lies on the straight line joining L and N. An object travels from L to M at an average speed of 10 m/s in 6 seconds and then from M to N at an average speed of 25 m/s. Find the average speed of the object for it's entire journey from L to N.
Answer:
16m/sStep-by-step explanation:
If the object travels from L to M at an average speed of 10 m/s in 6 seconds, we can get the distance LM using the formula;
Speed = Distance/Time
Distance = Speed * Time
Distance = 10 * 6
Distance = 60m
Hence LM = 60m
Since LN = LM + MN
160 = 60 + MN
MN = 160-60
MN = 100m
If the object moves from M to N at an average speed of 25 m/s througj a distance od 100m, the time taken will be expressed as;
Time = distance/speed
Time = 100/25
Time = 4secs
Total time taken to move from L to N is 6secs + 4secs = 10secs
The average speed of the object for it's entire journey from L to N will be:
Average speed = Distance LN/10
Average speed for the entire journey = 160/10 = 16m/s
Hence the average speed of the object for it's entire journey from L to N is 16m/s
Aubrey and Charlie are driving to a city that is 120 mi from their house. They have already traveled 20 mi, and they are driving at a constant rate of 50 mi/h. Complete the function that models the distance they drive as a function of time. Then complete a reasonable domain for this situation.
function notation in slope-intercept form: f(x) = ________
reasonable domain: ≤ x ≤
Answer: The function that models the distance they drive is
f(x) = 50x + 20 where x is the time in hours
reasonable domain: 0 ≤ x ≤ 3
Step-by-step explanation:
examples:
95 = 50(1.5) + 20 After driving another hour and a half, they will have driven a total of 95 miles.
120 = 50(2) + 20 This means that after 2 more hours they will reach their destination.
There is a little ambiguity in the question. The function could be written as if they are starting out. f(x) = 50t
20 = 50(.4) At 50 mph it took .4 hours to go 20 miles.
120 = 5(2.4) The whole trip took 2.4 hours.
Question 10 multiple choice
Answer: Choice B
(x-1)(x^3+x^2+5x+6)
==========================================
Explanation:
The 1 in the upper left box means that x = 1 is a root of the original polynomial.
So this means x-1 is a factor of the original polynomial.
This is because x = 1 leads to x-1 = 0 after subtracting 1 from both sides.
-----------------
The 0 in the last position of the bottom row shows we got a remainder of 0.
Getting a remainder 0 tells us that (x-1) is a factor of the polynomial. This synthetic division table confirms our initial guess.
The other values in that bottom row (1, 1, 5, 6) form coefficients to the polynomial 1x^3+1x^2+5x+6, or simply x^3+x^2+5x+6
-------------------
So we know that (x-1) and (x^3+x^2+5x+6) are factors
Meaning that,
x^4+4x^2+x-6 = (x-1)(x^3+x^2+5x+6)
You can confirm this by expanding out the right hand side (distribution rule).
55/5 divided by 11/4
Answer:
1/4 in fraction form or in decimal form its 0.25
Step-by-step explanation:
Answer: 4
Explanation: To solve (⁵⁵⁄₅) ÷ (¹¹⁄₄), you must follow PEMDAS. And so, I'd divide what's inside the parentheses. 55 divided by 5 is 11, and 11 divided by 4 is 2.75. Now, our equation should look like this: 11 ÷ 2.75. After dividing, you should get a quotient of 4.
I hope this helped!
Good luck <3
The submarine dove 150ft. from periscop height to an elevation of 200 ft. below sea level
which point is a representation of periscope height.
A) point A
B) point B
C) point C
D) point D
Answer:
c
Step-by-step explanation:
Help Geometry im very stupid
Its an obtuse triangle because one of the angle n is 98 degrees
Answer:
Obtuse Triangle
Step-by-step explanation:
First we have to find the measure of all the angles. We know that the sum of all angles in a triangles is 180 degrees.
So we add 35 and 47 which is 82 degrees.
Then we subtract 82 from 180 which is 98 degrees. Now we know that angle N is 82 degrees.
We can go ahead eliminate answer choices acute equiangular triangle (that only applies if all angles are 60 degrees) and right triangle (must have a right angle which is 90 degrees).
That now leaves us with acute and obtuse triangle. An acute triangle has angles that are less that 90 degrees. So that only leaves us with the answer choice obtuse triangle, because it has an angle of 98 degrees which is an obtuse angle.
So your answer is Obtuse Triangle.
Solve the inequality. Graph the solution.
3d−7d+2.8<5.8−27
Answer:
Step-by-step explanation:
Z>6
The solution to the given inequality would be d > 6, which is shown in the graph with the red line.
Given that,
The inequality to solve,
3d - 7d + 2.8 < 5.8 - 27
Used the concpet of expression for solving by using combining like terms and applying the operations addition, subtraction, etc.
Here, the inequality is,
3d - 7d + 2.8 < 5.8 - 27
Combine like terms,
- 4d + 2.8 < 5.8 - 27
Subtract 2.8 on both sides,
- 4d < 5.8 - 2.8 - 27
- 4d < 3 - 27
- 4d < - 24
Multiply both sides by - 1,
4d > 24
It can be written as,
4d - 24 > 0
4 (d - 6) > 0
d - 6 > 0
Therefore, the solution to an inequality is,
d > 6
And, the graph of solution d > 6 is shown in the attached image.
To learn more about inequality visit:
https://brainly.com/question/25944814
#SPJ6
how do you find the circumference of a circle if the diameter is 4
Answer: if your looking for an answer it’s c= 12.57
Step-by-step explanation:
What is 65% of 1150?
O 17.7
O 74.8
O 747.5
0 1769.2
Answer:
747.5
Step-by-step explanation:
Jaycie has a VIP membership to a movie theater, so she pays 27 dollars a year and 6.00 for each movie she sees. Claire doesn't have a membership, so she pays 8.25 for each movie she sees. How many movies would they have to see in a year in order to pay the same amount?
Answer:
12 movies
Step-by-step explanation:
Let the number of movies they see be represented by x
Jaycie has a VIP membership to a movie theater, so she pays 27 dollars a year and 6.00 for each movie she sees.
$27 + $6 × x
= 27 + 6x
Claire doesn't have a membership, so she pays 8.25 for each movie she sees.
= $8.25 × x
= 8.25x
How many movies would they have to see in a year in order to pay the same amount?
Jaycie = Claire
27 + 6x = 8.25x
27 = 8.25x - 6x
27 = 2.25x
x = 27/2.25
x = 12 movies
Therefore, they have to see 12 movies in a year in order to pay the same amount.
If I subtract the square root of 10 and -4, my answer will be
root(10)+4
......
you cant simplify it
What is the solution to p+1/2 = 3p + 7/5? Help would be appreciated. Do your best pls
Answer:
p = -9
Step-by-step explanation:
So, we need to find the value of p.
Please refer to the ATTACHMENT.....I hope it will help.Help find the value of x
-2(4-x)=12x-3
Answer:
Step-by-step explanation:
distributive property
-8 + 2x = 12x - 3
add 3 to both sides
-5 + 2x = 12x
subtract 2x from both sides
-5 = 10x
divide by 10 for both sides
-5/10 = x
-0.5 = x
actually it's -1/2 or -0.5
sixty-six subtracted from a number p
Someone please help me
Answer:
x = -3/2
Step-by-step explanation:
The removable discontinuity is at the value of x where a denominator factor cancels a numerator factor, and those factors are zero.
[tex]p(x)=\dfrac{2x+3}{4x^2-9}=\dfrac{2x+3}{(2x+3)(2x-3)}=\dfrac{2x+3}{2x+3}\cdot\dfrac{1}{2x-3}\\\\p(x)=\dfrac{1}{2x-3}\qquad x\ne-3/2[/tex]
That cancellation occurs where 2x+3 = 0, at x=-3/2.
The function p(x) has a removable discontinuity when x = -3/2.
Could someone please help with this question?
Anna completes a test that had 120 questions for a math competition. Each correct answer is worth +4 points, and each wrong answer is worth -2 points. Anna finishes the entire test and receives a score of zero. How many correct and incorrect answers did she have?
Answer:
40 correct and 80 incorrect
Step-by-step explanation:
40*4=160
80*2=160
Answer: 40 Correct and 80 Incorrect
Step-by-step explanation: Every answer she gets correct, she can afford to get 2 answer wrong meaning the number of incorrect answers must be twice the amount of correct. 40(4)=160 80(-2)=160 which evens out to zero
Which angle number is supplementary to angle Answer:
Answer:
Angle 8
Step-by-step explanation:
Supplementary angles are angles that are equal to 180 degrees.
Which is a perfect square?
O 20
0 21
O 24
O 25
Answer:
25
Step-by-step explanation:
25
(5)² = 25 OR √25 = 5